LSAT and Law School Admissions Forum

Get expert LSAT preparation and law school admissions advice from PowerScore Test Preparation.

 Administrator
PowerScore Staff
  • PowerScore Staff
  • Posts: 8916
  • Joined: Feb 02, 2011
|
#25438
Complete Question Explanation

Main Point—SN. The correct answer choice is (E)

The structure of the argument in this stimulus is a bit confusing. It begins with a conditional statement that we need not diagram, because it turns out not to be part of the author’s argument. Instead, the first sentence merely introduces a topic, the geometric growth of Earth’s population. This continued growth will mean that in a few centuries, there will be about one person per square foot of Earth’s surface.

In the second sentence, the author uses the “some people say...” technique, providing the viewpoint of someone else, which the author will conclude is incorrect. Some people say that by a few centuries from now, humans will have learned to colonize other planets and, because of this ability, the geometric growth of the population will not be a problem.

As we anticipated, the author disagrees with this position, and concludes that colonizing other planets would be a temporary solution at best. The author points out that if the population doubles every 30 years, then if half the Earth’s population emigrated to Mars in 2500, a date within the “few centuries” timeline provided in the first sentence, the population would be back to the 2500 level by 2530. Because of this basic math, the author concludes colonization of other planets would only be a temporary solution.

The question stem tells us that this is a Main Point question. Our prephrase is that the author concludes the colonization of other planets would be only a temporary solution to the problem of geometric population growth. Although there were no conclusion indicator words to help us find the conclusion, the author gave us a hint by using the the “some people say...” technique.

Answer choice (A): This statement was a claim made to provide the factual background for the argument. It was neither a premise nor a conclusion.

Answer choice (B): The author does not make a prediction about whether overpopulation will persist. Instead, the author concludes the planetary emigration idea will not be a permanent solution.

Answer choice (C): This statement was a premise offered in support of the conclusion.

Answer choice (D): As with answer choice (B), this choice is incorrect, because the author does not make a prediction about what will occur.

Answer choice (E): This is the correct answer choice, because it is a restatement of the author’s conclusion.
 nlittle
  • Posts: 18
  • Joined: Sep 09, 2017
|
#41712
Hello,

I can see how E uses more real estate in the stimulus, but when I read this, my thought was that A was correct because "Some people" and on seemed to raise a possible objection to the author's first claim, only to critique it in the claim that followed. To me, it seemed that the last two claims ("Some people" and on) were stated just to strengthen the first, which I, therefore, took to be the conclusion.

Could my mistake have been that, even if the first sentence was in fact the conclusion, the stand alone conclusion may not always be the "main point" itself? Was E correct because it is more of a hybrid each statement (conclusion+critique of an objection), rather than what I took to be a stand alone conclusion? Or perhaps, if the conclusion becomes qualified/distorted by other statements in some way, that those elements should be reflected in the correct answer choice? If this were the case I could see E being correct, but I just want to be sure I'm clear on the relationship between the mechanics of the stimulus and what the question stem is actually tasking me with.

Best,

Nick
 Adam Tyson
PowerScore Staff
  • PowerScore Staff
  • Posts: 5153
  • Joined: Apr 14, 2011
|
#41746
The key here, Nick, is to recognize that the "main point" or "main conclusion" of an argument is the thing that is getting all the support and giving none. Looking at the argument as a whole, what is the author trying to prove? In this case, is the whole argument designed to support the claim about how many people there will be? Is there, in fact, ANY evidence to support what the population will be or how much space we will take up? None - it's all just supposition - "if" this happens and "if" that happens.

That's why A is not the conclusion/main point here - there is no support for it, and the author isn't trying to prove it. Instead, he uses that as background information to lead into the claims that "some people say" something, but those people are wrong, and there's some evidence to support the claim that they are wrong.

Now, that structure - "some people say something, but they are wrong, and here's why" - is a VERY common LR stimulus structure, and it also forms the basis for many RC passages, and when you encounter it you will find that the main conclusion is almost always the "those people are wrong" statement. In this case, that matches up to "this would be a temporary solution at best", and that is how you know that is the main point here. The thing that some people say is the thing that the argument is designed to discredit; the "here's why" portion is the set of premises; the "they are wrong" portion is the conclusion.

Many arguments on the LSAT have multiple conclusions, and your task is to pick the one that is getting all the support and giving none. When you have two conclusions, one of them supports the other in some way. The one GETTING the support is the main conclusion; the one GIVING the support is not. Instead, we call that an "intermediate" or "subordinate" conclusion.

I hope that helps! Keep up the good work!
 Kellyg
  • Posts: 14
  • Joined: Jan 23, 2018
|
#46117
Hi,

I was stuck between B and E. I see how E is correct, but your explanation of why B is incorrect states "Answer choice (B): The author does not make a prediction about whether overpopulation will persist. Instead, the author concludes the planetary emigration idea will not be a permanent solution." ....but when taken with the rest of his argument would overpopulation persisting not be a sufficient condition for the idea that emigration will not be a permanent solution making B plausibly correct? Further, I felt B was more directly stated in the stimulus which was also why I chose it. Any suggestions as to where my reasoning is going wrong?
User avatar
 Jonathan Evans
PowerScore Staff
  • PowerScore Staff
  • Posts: 726
  • Joined: Jun 09, 2016
|
#46762
Hi, Kelly!

Good question! The author makes no definite prediction about whether the geometric population growth will continue. The author discusses what will happen if it continues. Answer choice (B) states that the problem of overpopulation will likely persist. The author does not reach this conclusion with this degree of likelihood/certainty. The author's statements indicate that this problem will persist if geometric population growth persists.

There is in fact a direct statement in the stimulus that is a close paraphrase of the statement in answer choice (E). The author states, " This would [...] be a temporary solution at best." "This" refers to the proposals to colonize other planets. Put together, the author concludes: "Proposals to colonize other planets would be at best a temporary solution to the problem of overpopulation." This is a very close paraphrase of answer choice (E), and thus, this is the credited response.

I hope this helps!

Get the most out of your LSAT Prep Plus subscription.

Analyze and track your performance with our Testing and Analytics Package.